Yahoo Answers is shutting down on May 4th, 2021 (Eastern Time) and beginning April 20th, 2021 (Eastern Time) the Yahoo Answers website will be in read-only mode. There will be no changes to other Yahoo properties or services, or your Yahoo account. You can find more information about the Yahoo Answers shutdown and how to download your data on this help page.

Integral of Inverse Trig Function?

Calculate the integral ∫arctan(x) dx.

I know the answer because I can look it up in the table of integrals at the back of a calculus book, but I am drawing a blank trying to work it out.

5 Answers

Relevance
  • Puggy
    Lv 7
    1 decade ago
    Favorite Answer

    Integration by parts, followed by U-substitution.

    Since I know you're an expert I'll just not explain each step and do it.

    ∫(arctan(x) dx)

    Let u = arctan(x). dv = dx

    du = 1/[1 + x^2] dx. v = x

    x arctan(x) - ∫ (x/[1 + x^2] dx )

    x arctan(x) - ∫ (1/[1 + x^2] x dx )

    Let u = 1 + x^2.

    du = 2x dx

    (1/2) du = x dx

    x arctan(x) - ∫ ( (1/u) (1/2) du )

    x arctan(x) - (1/2) ∫ ( (1/u) du )

    x arctan(x) - (1/2)ln|u| + C

    x arctan(x) - (1/2)ln|1 + x^2| + C

    x arctan(x) - (1/2)ln(1 + x^2) + C

  • 1 decade ago

    The complex exponential version of ArcTan(x) is:

    (1/2) i (Log(1 - i x) - Log(1 + i x))

    The integral of this is

    (1/2) x i (Log(1 - i x) - Log(1 + i x)) - (1/2) Log(1 + x²)

    But the first two terms is really x ArcTan(x), so that the answer is:

    x ArcTan(x) - (1/2) Log(1 + x²)

  • Zach
    Lv 5
    1 decade ago

    Wikipedia has a section in their article called "Indefinite integrals of inverse trigonometric functions"

    It has a proof using Integration By Parts method for arcsin. This would be identical to yours.

  • ted s
    Lv 7
    1 decade ago

    Hint: IBP with u = arctan x , dv = dx.....( x arctan x - [1/2] ln { x² +1} + C)

  • How do you think about the answers? You can sign in to vote the answer.
  • Anonymous
    5 years ago

    because of the fact you calculated it by using hand incorrectly. If I re-differentiate your consequence, I finally end up with: (2*x^2 - (x^2 + 25)*ln(x^2 + 25))/(2*x^4 + 50*x^2) If I re-differentiate the TI-89 calculator's consequence, I finally end up with: a million/(x^2+25)

Still have questions? Get your answers by asking now.